L'Hopital's Rule

Read this section to learn how to use and apply L'Hopital's Rule. Work through practice problems 1-3.

Other "Indeterminate Forms"

" 0 / 0 " is called an indeterminate form because knowing that \mathrm{f} approaches 0 and \mathrm{g} approaches 0 is not enough to determine the limit of \mathrm{f} / \mathrm{g}, even if it has a limit. The ratio of a "small" number divided by a "small" number can be almost anything as the three simple " 0/0 " examples show:

\lim \limits_{x \rightarrow 0} 3 x / x=3, \lim \limits_{x \rightarrow 0} x^{2} / x=0, and  \lim \limits_{x \rightarrow 0} 5 x / x^{3}=\infty

Similarly, " \infty/\infty " is an indeterminate form because knowing that \mathrm{f} and \mathrm{g} both grow arbitrarily large is not enough to determine the value limit of \mathrm{f} / \mathrm{g} or if the limit exists:

\lim \limits_{x \rightarrow \infty} 3 x / x=3, \lim \limits_{x \rightarrow \infty} x^{2} / x=\infty, and \lim \limits_{x \rightarrow \infty} 5 x / x^{3}=0

Besides the indeterminate quotient forms " 0/0 " and " \infty/\infty " there are several other "indeterminate forms". In each case, the resulting limit depends not only on each function's limit but also on how quickly each function approaches its limit.

Product: If f approaches 0, and g grows arbitrarily large, the product \mathrm{f} \cdot \mathrm{g} has the indeterminant form " 0 \cdot \infty ".

Exponent: If \mathrm{f} and \mathrm{g} both approach 0, the function \mathrm{f}^{\mathrm{g}} has the indeterminant form " 0^{0} ".

If f approaches 1, and g grows arbitrarily large, the function \mathrm{f}^{\mathrm{g}} has the indeterminant form " 1^{\infty} ".

If f grows arbitrarily large, and g approaches 0, the function f^{g} has the indeterminant form " \infty^{0} ".

Difference: If \mathrm{f} and \mathrm{g} both grow arbitrarily large, the function \mathrm{f}-\mathrm{g} has the indeterminant form " \infty - \infty ".

Unfortunately, l'Hô pital's Rule can only be used directly with an indeterminate quotient (" 0/0 " or " \infty/\infty '), but these other forms can be algebraically manipulated into quotients, and then l'Hô pital's Rule can be applied to the resulting quotient.

Example 5: Evaluate \lim \limits_{x \rightarrow 0^{+}} \mathrm{x} \cdot \ln (\mathrm{x}) (" 0 \cdot(-\infty) " form)

Solution: This limit involves an indeterminate product, and we need a quotient in order to apply l'Hô pital's Rule. We can rewrite the product x \cdot \ln (x) as the quotient \frac{\ln (x)}{1 / x}, and then so apply l'Hô pital's Rule

\begin{aligned} \lim \limits_{x \rightarrow 0^{+}} x \ln (x) &=\lim \limits_{x \rightarrow 0^{+}} \frac{\ln (x)}{1 / x} \rightarrow \frac{\infty}{\infty} \\ &=\lim \limits_{x \rightarrow 0^{+}} \frac{1 / x}{-1 / x^{2}}=\lim \limits_{x \rightarrow 0^{+}}-x=0 \end{aligned}

A product f \cdot g with the indeterminant form " 0 \cdot \infty " can be rewritten as a quotient, \frac{f}{1 / g} or \frac{g}{1 / f}, and then l'Hô pital's Rule can be used.

Example 6: Evaluate \lim \limits_{x \rightarrow 0^{+}} \mathrm{x}^{\mathrm{x}} ( 0^{0}  form)

Solution: An indeterminate exponent can be converted to a product by recalling a property of exponential and
logarithm functions: for any positive number a, a=e^{\ln (a)} so f^{g}=e^{\ln (f g)}=e^{g \ln (f)}.
\lim \limits_{x \rightarrow 0^{+}} x^{x}=\lim \limits_{x \rightarrow 0^{+}} e^{\ln \left(x^{x}\right)}=\lim \limits_{x \rightarrow 0^{+}} e^{x \cdot \ln (x)} and this last limit involves an indeterminate product \mathrm{x} \cdot \ln (\mathrm{x}) \rightarrow 0 \cdot(-\infty) which we converted to a quotient and evaluated to be 0 in Example 5.
Our final answer is then \mathrm{e}^{\mathbf{0}}=\mathbf{1}:

\lim \limits_{x \rightarrow 0^{+}} x^{x}=\lim \limits_{x \rightarrow 0^{+}} e^{\ln \left(x^{x}\right)}=\lim \limits_{x \rightarrow 0^{+}} e^{x \cdot \ln (x)}=e^{0}=1

An indeterminate form involving exponents, \mathrm{f}^{\mathrm{g}} with the form " 0^{0}, " " 1^{\infty}, " or " \infty ^{0}, " can be converted to an indeterminate product by recognizing that \mathrm{f}^{\mathrm{g}}=\mathrm{e}^{\mathrm{g} \cdot \ln (\mathrm{f})} and then determining the limit of g \cdot \ln (f). The final result is \mathrm{e}^{(\text {limit of } \mathrm{g} \cdot \ln (\mathrm{f}))}.

Example 7: Evaluate \lim \limits_{x \rightarrow \infty}\left(1+\frac{a}{x}\right)^{x} (" 1^{\infty} " form)

Solution: \lim \limits_{x \rightarrow \infty}\left(1+\frac{a}{x}\right)^{x}=\lim \limits_{x \rightarrow \infty} \mathrm{e}^{\mathrm{x} \cdot \ln (1+\mathrm{a} / \mathrm{x})} so we need \lim \limits_{x \rightarrow \infty} \mathrm{x} \cdot \ln \left(1+\frac{\mathrm{a}}{\mathrm{x}}\right) \lim \limits_{x \rightarrow \infty} x \cdot \ln \left(1+\frac{a}{x}\right) \rightarrow " \infty \cdot 0 "  an indeterminate product so rewrite it as a quotient

=\lim \limits_{x \rightarrow \infty} \frac{\ln \left(1+\frac{a}{x}\right)}{1 / x} \rightarrow  \frac{0}{0} an indeterminate quotient so use l'Hô pital's Rule

=\lim \limits_{x \rightarrow \infty} \frac{\left(\frac{-a / x^{2}}{1+\frac{a}{x}}\right)}{\left(-\frac{1}{x^{2}}\right)}=\lim \limits_{x \rightarrow \infty} \frac{a}{1+\frac{a}{x}} \rightarrow \frac{\mathrm{a}}{1}=\mathbf{a}

Finally, \lim \limits_{x \rightarrow \infty}\left(1+\frac{a}{x}\right)^{x}=\lim \limits_{x \rightarrow \infty} e^{x \cdot \ln (1+a / x)}=\mathrm{e}^{\mathbf{a}}.